6
$\begingroup$

It is known that ILP with a fixed number of variables is in $\mathsf{P}$[Les83]. (For convenience, the decision version of the problem is given in the post.)

I have the following two questions regarding it.

Q1. Where does it lie within $\mathsf{P}$?

Q2. Is it $\mathsf{P}$-hard, or is it in $\mathsf{NC}$?


[The decision problem]

Input: Let $m,n\in \mathbb{Z^+}$, $A$ is ${m\times n}$ matrix with intgeral coefficients, and $b\in \mathbb{Z}^m$.

Promise: The value of $n$ will be kept fix. It won't change from one instance of the problem to another.

Goal: Does there exist a vector $x\in \mathbb{Z}^n$ that satisfies the set of inequalities $Ax\leq b$?

Additionally, the size of an instance is defined as $m\cdot n\cdot \log(a+2)$, where $a$ is the maximum magnitude of the entries of the matrix $A$ and the vector $b$.


My attempt:

Remark 1: I don't see any (obvious) way to reduce Linear programming to it. The key hurdle is that the restriction on $n$ is removed for LP. (This is my failed attempt to show $\mathsf{P}$ hardness.)

Remark 2: In [Les83], there is a use of the LLL algorithm (a lattice basis reduction algorithm) as a subroutine, which is a sequential algorithm [See page 3 of this]. (This is my failed attempt to show containment in $\mathsf{NC}$.)

Emil Jeřábek
20.6k3 gold badges70 silver badges101 bronze badges
asked Oct 30 at 16:43
$\endgroup$
0

0

Know someone who can answer? Share a link to this question via email, Twitter, or Facebook.

Your Answer

Draft saved
Draft discarded

Sign up or log in

Sign up using Google
Sign up using Email and Password

Post as a guest

Required, but never shown

Post as a guest

Required, but never shown

By clicking "Post Your Answer", you agree to our terms of service and acknowledge you have read our privacy policy.

Start asking to get answers

Find the answer to your question by asking.

Ask question

Explore related questions

See similar questions with these tags.